Incompleteness of SOL

Dominik Kirst kirst at cs.uni-saarland.de
Wed Sep 8 09:20:00 EDT 2021


Thanks a lot for the answers, Section 7.7 in Neil’s book was exactly the type of comment I was looking for.

Best wishes,
Dominik



> On 8. Sep 2021, at 13:22, Tennant, Neil <tennant.9 at osu.edu> wrote:
> 
> Dear Dominik,
> The answers to your questions are positive. 
> Please see my book Natural Logic, Edinburgh University Press, 1978. 
> Best regards,
> Neil Tennant
> 
> Get Outlook for iOS <https://aka.ms/o0ukef>
> From: FOM <fom-bounces at cs.nyu.edu> on behalf of Richard Kimberly Heck <richard_heck at brown.edu>
> Sent: Tuesday, September 7, 2021 6:28:35 PM
> To: Dominik Kirst <kirst at cs.uni-saarland.de>; fom at cs.nyu.edu <fom at cs.nyu.edu>
> Subject: Re: Incompleteness of SOL
>  
> On 9/7/21 7:54 AM, Dominik Kirst wrote:
> > Dear FOM-members,
> >
> > I have a question regarding the incompleteness of second-order logic: in textbooks (e.g. Shapiro 1991) the incompleteness of sound and effective deduction systems for SOL is usually obtained as a consequence of Gödel’s incompleteness theorem. However, it is usually also shown that, for instance due to the categoricity of second-order PA, the standard semantics for SOL fails to be compact. Therefore trivially no sound deduction system (compact by the convention that T |- A if T’ |- A for some finite T’ <= T) can be complete, at least in the strong sense that T |= A would imply T |- A for all A and possibly infinite T.
> 
> Yes, though I would have thought that the very idea of a 'proof from
> infinitely many premises' is somewhat artificial when we're working in a
> conventional sort of system. So we can only say that you have such a
> proof when you have one from finitely many premises. So this ends up
> being a somewhat weak reason to say that the logic is incomplete: It's
> incomplete in cases where you don't really have a sensible notion of
> proof anyway. In particular, it leaves open the possibility that we
> should have T |= A iff T |- A for all finite sets T, which would be a
> pretty significant fact were it only true.
> 
> 
> > Has this argument been explicitly mentioned in the literature and compared to the more heavy-weight argument invoking Gödel’s incompleteness? And is the reason for the latter being the standard proof that it also refutes the weaker form of completeness where |= A implies |- A for all A?
> 
> Which is of course equivalent to the finite sets version, given the
> deduction theorem.
> 
> Riki
> 
> 

-------------- next part --------------
An HTML attachment was scrubbed...
URL: </pipermail/fom/attachments/20210908/057da0a2/attachment-0001.html>


More information about the FOM mailing list